Đến nội dung

Hình ảnh

Bất đẳng thức - Cực trị


  • Please log in to reply
Chủ đề này có 238 trả lời

#41
royal1534

royal1534

    Trung úy

  • Điều hành viên THCS
  • 773 Bài viết

 

 

Bài 24: Cho $x,y,z>0$

CM: $2(x^{2}+y^{2}+z^{2})+xyz+8\geq 5(a+b+c)$

 

 

 
Lời Giải: Sử dụng bất đẳng thức AM-GM, ta có
$$x+y+z=\frac{1}{3}.3.(x+y+z) \leq \frac{1}{6}[9+(x+y+z)^2].$$
Do đó ta chỉ cần chứng minh
$$12(x^2 + y^2 + z^2) + 6xyz + 48 \geq 5 [(x + y + z)^2 + 9] .$$
Bất đẳng thức này tương đương với
$$7(x^2 + y^2 + z^2) + 6xyz + 3 \geq 10(xy + yz + zx),$$
$$4(x^2 + y^2 + z^2 - xy - yz - zx) + 3 [x^2 + y^2 + z^2 + 2xyz +1 - 2(xy + yz + zx)] \geq 0,$$
 Ta có $$x^2 + y^2 + z^2 \geq xy + yz + zx$$  
Nên ta chỉ cần chứng minh:
$$x^2 + y^2 + z^2 + 2xyz +1 \geq 2(xy + yz + zx)$$ 
Mặt khác đây là 1 kết quả đúng theo bài 21
Bài toán được chứng minh xong. Đẳng thức xảy ra khi và chỉ khi $ x= y = z = 1$


#42
Trung Kenneth

Trung Kenneth

    Trung sĩ

  • Thành viên
  • 100 Bài viết

Bai 26: Cho a,b,c > 0

CMR : $\frac{a}{a+b}+\frac{b}{b+c}+\frac{c}{c+a}\geq \frac{3}{2}$

Bai 27: Cho x,y,z >0  t/m $x^2+y^2+z^2=1$

Tim min : $\frac{x}{y^2+z^2}+\frac{y}{z^2+x^2}+ \frac{z}{x^2+y^2}$



#43
NTA1907

NTA1907

    Thượng úy

  • Thành viên
  • 1014 Bài viết

Bai 26: Cho a,b,c > 0

CMR : $\frac{a}{a+b}+\frac{b}{b+c}+\frac{c}{c+a}\geq \frac{3}{2}$

Bđt sai khi $a=3, b=4, c=5$. Mình nghĩ bài này cần 1 đk về tổng hoặc tích của $a,b,c$ nữa


Vũ trụ không có biên trong không gian, không có bắt đầu và kết thúc trong thời gian và chẳng có việc gì cho đấng sáng thế phải làm ở đây cả.

 


#44
PlanBbyFESN

PlanBbyFESN

    Thiếu úy

  • Điều hành viên OLYMPIC
  • 637 Bài viết
 

Bài 19: Cho $a^2+b^2+c^2=3$ a,b,c>0

CMR: $\frac{ab}{3+c^2}+\frac{bc}{3+a^2}+\frac{ca}{3+b^2}\leq \frac{3}{4}$

Cách 3 cho bài trên:

Ta sẽ có đpcm nếu có $\frac{bc}{3+a^{2}}\leq \frac{3}{4}(\frac{bc(b+c)}{ab(a+b)+bc(b+c)+ca(c+a)})$  (*)

Thật vậy:

$(*)\Leftrightarrow 3(3+a^{2})(b+c)\geq 4\sum ab(a+b)$

Thay 3=a2+b2+c2 vào : $\Leftrightarrow 3(2a^{2}+b^{2}+c^{2})(b+c)\geq 4\sum ab(a+b)$

Bằng phân tích ghép bình phương:

$\Leftrightarrow 2c(a-c)^{2}+2b(a-b)^{2}+(b+c)(b-c)^{2}\geq 0$

Luôn đúng $\Rightarrow$ ĐPCM

Dấu "=" là a=b=c=1

 

Bđt sai khi $a=3, b=4, c=5$. Mình nghĩ bài này cần 1 đk về tổng hoặc tích của $a,b,c$ nữa

Cứ làm đi, đề đúng đấy . Gợi ý: Giả sử: $a\geq b\geq c$

 

P/S: 1h đi học rồi, tối sẽ làm tiếp topic và 1 số bài tập dạng dễ hơn và một số bài dạng... bó tay hơn mà thú vị hơn  >:) 


Bài viết đã được chỉnh sửa nội dung bởi PlanBbyFESN: 22-01-2016 - 12:51

:huh:


#45
tpdtthltvp

tpdtthltvp

    Trung úy

  • Điều hành viên THCS
  • 831 Bài viết

Bai 26: Cho a,b,c > 0

CMR : $\frac{a}{a+b}+\frac{b}{b+c}+\frac{c}{c+a}\geq \frac{3}{2}$

 

 

Bđt sai khi $a=3, b=4, c=5$. Mình nghĩ bài này cần 1 đk về tổng hoặc tích của $a,b,c$ nữa

Điều kiện bài này là: $a\geq b\geq c$

Ta có:

$\Leftrightarrow (\frac{a}{a+b}-\frac{1}{2})+(\frac{b}{b+c}-\frac{1}{2})+(\frac{c}{c+a}-\frac{1}{2})\geq 0\Leftrightarrow \frac{a-b}{2(a+b)}+\frac{b-c}{2(b+c)}+\frac{c-a}{2(c+a)}\geq 0\Leftrightarrow \frac{a-b}{2}(\frac{1}{a+b}-\frac{1}{b+c})+\frac{c-a}{2}(\frac{1}{c+a}-\frac{1}{b+c})\geq 0\Leftrightarrow \frac{(a-b)(c-a)}{2(a+b)(b+c)}+\frac{(c-a)(b-a)}{2(c+a)(b+c)}\geq 0\Leftrightarrow \frac{(a-b)(a-c)(b-c)}{2(a+b)(b+c)(c+a)}\geq 0$ , đúng.


$\color{red}{\mathrm{\text{How I wish I could recollect, of circle roud}}}$

$\color{red}{\mathrm{\text{The exact relation Archimede unwound ! }}}$

 


#46
PlanBbyFESN

PlanBbyFESN

    Thiếu úy

  • Điều hành viên OLYMPIC
  • 637 Bài viết
 

Bài 24: Cho $x,y,z>0$

CM: $2(x^{2}+y^{2}+z^{2})+xyz+8\geq 5(a+b+c)$

Bài 25: Cho a,b,c là 3 cạnh của tam giác :

CM: $\sum \frac{a}{b+c}<2$

 

Bài 24:        

Xét $x-1;y-1;z-1$ luôn tồn tại hai bé cùng dấu nên theo nguyên tắc đi dép lê luôn tồn tại hai số có tích lớn hơn hoặc bằng 0.

 

Giả sử $z(x-1)(y-1)\geq 0\Rightarrow xyz\geq xz+yz-z$ 

Đưa bài toán về chứng minh:

$xz+yz-z+2(x^{2}+y^{2}+z^{2})+8\geq 5(x+y+z)$

 

Thực ra là: $\Leftrightarrow (y+z-2)^{2}+(x+z-2)^{2}+3(x-1)^{2}+3(y-1)^{2}+2(z-1)^{2}\geq 0$

Dấu "=" xảy ra $\Leftrightarrow x=y=z=1$

Bài 25: Một bài cơ bản


:huh:


#47
PlanBbyFESN

PlanBbyFESN

    Thiếu úy

  • Điều hành viên OLYMPIC
  • 637 Bài viết

Bài 24: Cho $x,y,z>0$

CM: $2(x^{2}+y^{2}+z^{2})+xyz+8\geq 5(a+b+c)$

 

Bài 24: Cách giải thứ 3 cho bài 24 (tình cờ gặp ở pqr) dành cho những ai "ham" Schur

 

Lời giải: (T. Trần Nam Dũng)

AM-GM:

$abc+\frac{1}{2}=\frac{abc+abc+1}{2}\geq \frac{3abc}{2\sqrt[3]{abc}}\geq \frac{9abc}{2(a+b+c)}$

và:

$5(a+b+c)=\frac{5}{6}.2.3(a+b+c)\leq \frac{5}{6}\left [ 3^{2}+(a+b+c)^{2} \right ]$

Vậy nên chỉ cần CM:

$2(a^{2}+b^{2}+c^{2})+\frac{9abc}{2(a+b+c)}+\frac{15}{2}\geq \frac{5}{6}(9+(a+b+c)^{2})$

hay $7(a^{2}+b^{2}+c^{2})+\frac{27abc}{a+b+c}\geq 10(ab+bc+ca)$

 

Mà $4(a^{2}+b^{2}+c^{2})\geq 4(ab+bc+ca)$

nên BĐT đúng $\Leftrightarrow a^{2}+b^{2}+c^{2}+\frac{9abc}{a+b+c}\geq 2(ab+bc+ca)$

Schur bậc 3 phân thức  (ĐPCM)

 

Dấu bằng xảy ra $\Leftrightarrow a=b=c=1$

 


:huh:


#48
PlanBbyFESN

PlanBbyFESN

    Thiếu úy

  • Điều hành viên OLYMPIC
  • 637 Bài viết

Tiếp theo phần vận dụng nguyên tắc Đirichlê ta sẽ tiếp tục tập trung vào đậm chất THCS: AM-GM, Cô si ngược, Cauchy-Schwarz, bđt phụ....

 

Bài 28(*): Cho $a,b,c>0; a+b+c=3$

CM: $\frac{1}{a^{2}+b^{2}+2}+\frac{1}{b^{2}+c^{2}+2}+\frac{1}{c^{2}+a^{2}+2}\leq \frac{3}{4}$

 

Bài 29: $x,y,z>0; xyz=1$

CM: $\frac{1}{x^{2}+x+1}+\frac{1}{y^{2}+y+1}+\frac{1}{z^{2}+z+1}\geq 1$

 

Bài 30: $a,b,c>0; abc=1$

CM: $\frac{a}{2a^{3}+1}+\frac{b}{2c^{3}+1}+\frac{c}{2a^{3}+1}\leq 1$

 

Bài 31: $a,b,c>0$

CM: $\frac{a^{3}+b^{3}+c^{3}}{3abc}+(\frac{ab+bc+ca}{a^{2}+b^{2}+c^{2}})^{2}\geq 2$

 

P/S: lưu ý bài 27 chưa giải

À mà các bài đã giải mà ai có lời giải có ý tưởng mới mẻ thú vị thì cứ đăng, đừng ngại!  @};-  @};-  @};- Toán đẹp phải mò được nhiều cách :)))


Bài viết đã được chỉnh sửa nội dung bởi PlanBbyFESN: 22-01-2016 - 21:23

:huh:


#49
Gachdptrai12

Gachdptrai12

    Thượng sĩ

  • Điều hành viên THCS
  • 280 Bài viết

bài 28 

bđt tương dương 

$\sum \frac{a^{2}+b^{2}}{a^{2}+b^{2}+2}\geq \frac{3}{2}\Leftrightarrow$

mà $a^{2}+b^{2}=\frac{(a+b)^{2}+(a-b)^{2}}{2}$

bđt tương đương$\sum \frac{(a+b)^{2}}{a^{2}+b^{2}+2}+\sum \frac{(a-b)^{2}}{a^{2}+b^{2}+2}\geq 3$

áp dụng bđt C-S$\sum \frac{(a+b)^{2}}{a^{2}+b^{2}+2}\geq \frac{((a+b)+(b+c)+(c+a))^{2}}{\sum (a^{2}+b^{2}+2)}$ và 

$\sum \frac{(a-b)^{2}}{a^{2}+b^{2}+2}\geq \frac{((a-b)+(b-c)+(a-c))^{2}}{\sum (a^{2}+b^{2}+2)}$

đên đây chỉ cần tương đường thì ra (a-b)(b-c) >=0 tương tự thì ra (a-c)(b-c)>=0 và (a-B(b-c)>=0 và tích của 3 đại ượng lớn hơn 0 nên tồn tại một đại lượng lớn hơn 0 =>dpcm đây là bài iran 2009 và cách yếu tố ít nhất của anh cẩn 

p/s lần sau sẽ ko bao h` dùng sigma nữa


Bài viết đã được chỉnh sửa nội dung bởi Gachdptrai12: 22-01-2016 - 22:45


#50
royal1534

royal1534

    Trung úy

  • Điều hành viên THCS
  • 773 Bài viết

Tiếp theo phần vận dụng nguyên tắc Đirichlê ta sẽ tiếp tục tập trung vào đậm chất THCS: AM-GM, Cô si ngược, Cauchy-Schwarz, bđt phụ....

 

Bài 28(*): Cho $a,b,c>0; a+b+c=3$

CM: $\frac{1}{a^{2}+b^{2}+2}+\frac{1}{b^{2}+c^{2}+2}+\frac{1}{c^{2}+a^{2}+2}\leq \frac{3}{4}$

 

 

Xin trích lại 1 lời giải của anh Cẩn

------------------
Viết lại bđt cần chứng minh:
 
$$\sum \frac{1}{a^{2}+b^{2}+2} \leq \frac{3}{4}$$
 
$$\Leftrightarrow \frac{1}{2}-\sum \frac{1}{a^{2}+b^{2}+2} \leq \frac{3}{4}$$
 
$$\Leftrightarrow \sum \frac{a^{2}+b^{2}}{a^{2}+b^{2}+2} \geq \frac{3}{2}$$
 
Ta có đánh giá $$a^{2}+b^{2}=\frac{(a+b)^{2}+(a-b)^{2}}{2}$$
 
Nên bđt cần chứng minh $$\leftrightarrow \sum \frac{(a+b)^{2}}{a^{2}+b^{2}+2}+\sum \frac{(a-b)^{2}}{a^{2}+b^{2}+2} \geq 3$$
 
Sử dụng bất đẳng thức Cauchy-Swarchz ta có 
$$\sum \frac{(a+b)^{2}}{a^{2}+b^{2}+2} \geq \frac{4(a+b+c)^{2}}{2(a^{2}+b^{2}+c^{2})+6}$$
 
$$\sum \frac{(a-b)^{2}}{a^{2}+b^{2}+2} \geq \frac{(|a-b|+|b-c|+|a-c|)^{2}}{2(a^{2}+b^{2}+c^{2})+6}=\frac{4(a-c)^{2}}{2(a^{2}+b^{2}+c^{2})+6}$$
Ta đưa bất đẳng thức về chứng minh:
 
$$\leftrightarrow \frac{4(a+b+c)^{2}+4(a-c)^{2}}{2(a^{2}+b^{2}+c^{2})+6} \geq 3$$
 
$$\leftrightarrow 2(a+b+c)^{2}+2(a-c)^{2} \geq 3(a^{2}+b^{2}+c^{2}+3)$$
 
Sử dụng giả thuyết $a+b+c=3$.BĐT cần chứng minh:
 
$$\leftrightarrow (a+b+c)^{2}+2(a-c)^{2} \geq 3(a^{2}+b^{2}+c^{2})$$
 
$$\leftrightarrow (a-b)(b-c) \geq 0 (1)$$
 
Nhận thấy bđt trên không phải luôn đúng.Nhưng  ta có thể ''ép'' nó đúng:
 
Thật vậy.Thiết lập các đánh giá tương tự ta có thể đưa bất đẳng thức về chứng minh:$$(c-a)(b-c) \geq 0 (2)$$ hoặc $$(c-a)(a-b) \geq 0 (3)$$
 
Vậy nếu trong $(1),(2),(3)$ có 1 bất đẳng thức đúng thì bài toán được chứng minh.Nhận thấy $$[(a-b)(b-c)][(b-c)(c-a)][(c-a)(a-b)]=(a-b)^{2}(b-c)^{2}(c-a)^{2} \geq 0$$
 
$\rightarrow$ Trong 3 đại lượng $[(a-b)(b-c)];[(b-c)(c-a)];[(c-a)(c-b)]$ có ít nhất 1 đại lượng không âm.Phép chứng minh hoàn tất.Đẳng thức xảy ra khi $a=b=c=1$
 


#51
PlanBbyFESN

PlanBbyFESN

    Thiếu úy

  • Điều hành viên OLYMPIC
  • 637 Bài viết

Bài 29: $x,y,z>0; xyz=1$

CM: $\frac{1}{x^{2}+x+1}+\frac{1}{y^{2}+y+1}+\frac{1}{z^{2}+z+1}\geq 1$

 

Bài 30: $a,b,c>0; abc=1$

CM: $\frac{a}{2a^{3}+1}+\frac{b}{2c^{3}+1}+\frac{c}{2a^{3}+1}\leq 1$

Bài 29: 

Đặt $x=\frac{bc}{a^{2}};y=\frac{ca}{b^{2}};c=\frac{ab}{c^{2}}$

Ta có abc=1.

 

VT trở thành:

$\frac{a^{4}}{a^{4}+b^{2}c^{2}+a^{2}bc}+\frac{b^{4}}{b^{4}+c^{2}a^{2}+ab^{2}c}+\frac{c^{4}}{c^{4}+a^{2}b^{2}+abc^{2}}\geq \frac{(a^{2}+b^{2}+c^{2})^{2}}{a^{4}+b^{4}+c^{4}+a^{2}b^{2}+b^{2}c^{2}+c^{2}a^{2}+abc(a+b+c)}$

 

Kết hợp $abc(a+b+c)\leq (a^{2}b^{2}+b^{2}c^{2}+c^{2}a^{2})$ (AM-GM)

 

Ta sẽ có ĐPCM với điểm rơi a=b=c=1

P/s: Bài 29 nhiều ứng dụng bao gồm cả bài 30


:huh:


#52
royal1534

royal1534

    Trung úy

  • Điều hành viên THCS
  • 773 Bài viết

 

Bai 27: Cho x,y,z >0  t/m $x^2+y^2+z^2=1$

Tim min : $\frac{x}{y^2+z^2}+\frac{y}{z^2+x^2}+ \frac{z}{x^2+y^2}$

Ta có bđt phụ: 
Với mọi $1 \geq x>0$ ,BĐT sau đúng $$x(1-x^2)\leq \frac{2}{3\sqrt{3}}$$ 
Thật vậy 
$$BDT\Leftrightarrow x^2(1-x^2)^2\leq \frac{4}{27}$$
Áp dụng AM-GM ,ta có 
$$x^2(1-x^2)^2=\frac{1}{2}.2x^2.(1-x^2)(1-x^2)\leq \frac{1}{2}\left [ \frac{2x^2+(1-x^2)+(2-x^2)}{3} \right ]^3=\frac{4}{27}$$
------------------
Quay lại bài toán:
Dự đoán $Min$ của $VT=\frac{3\sqrt{3}}{2}$:
Ta có $x^2+y^2+z^2=1$ nên ta viết BĐT thành$$\frac{x}{1-x^2}+\frac{y}{1-y^2}+\frac{z}{1-z^2}\geq \frac{3\sqrt{3}}{2}(x^2+y^2+z^2)$$
Do đó,ta chỉ cần chứng minh $$\frac{x}{1-x^2}\geq \frac{3\sqrt{3}}{2}x^2$$ hay $$\frac{1}{x(1-x^2)}\geq \frac{3\sqrt{3}}{2}$$
Nhưng BĐT đó đã được giải quyết triệt để
Phép chứng minh hoàn tất.Đẳng thức xảy ra tại $x=y=z=\frac{1}{\sqrt{3}}$

Bài viết đã được chỉnh sửa nội dung bởi royal1534: 22-01-2016 - 22:09


#53
PlanBbyFESN

PlanBbyFESN

    Thiếu úy

  • Điều hành viên OLYMPIC
  • 637 Bài viết

Good solution, roy!

 

Bài 31: $a,b,c>0$

CM: $\frac{a^{3}+b^{3}+c^{3}}{3abc}+(\frac{ab+bc+ca}{a^{2}+b^{2}+c^{2}})^{2}\geq 2$

 

Bài 30 vận dụng bài 29 mời các thím.(nói là vận dụng chứ không đến nỗi dễ đâu)

----------------------------

 

Bài 31:

 

$\frac{a^{3}+b^{3}+c^{3}}{3abc}+(\frac{ab+bc+ca}{a^{2}+b^{2}+c^{2}})^{2}\geq 2\sqrt{\frac{(ab+bc+ca)^{2}(a^{3}+b^{3}+c^{3})}{3abc(a^{2}+b^{2}+c^{2})^{2}}}$

 

Bất đẳng thức này được suy ra trực tiếp từ:

$(ab+bc+ca)^{2}\geq 3abc(a+b+c)$  (AM-GM)

và:

$(a^{3}+b^{3}+c^{3})(a+b+c)\geq (a^{2}+b^{2}+c^{2})^{2}$

 

$\Rightarrow$  ĐPCM

 

--------------------------------------

 

Bài 32: $a,b,c>0$

CM: $\frac{a+b}{c}+\frac{b+c}{a}+\frac{c+a}{b}\geq \frac{a}{b+c}+\frac{b}{c+a}+\frac{c}{a+b}+\frac{9}{2}$

 

Bài 33: $a,b,c\in \left [ 1;2 \right ]$

CM: $(a+b+c)(\frac{1}{a}+\frac{1}{b}+\frac{1}{c})\geq 6(\frac{a}{b+c}+\frac{b}{c+a}+\frac{c}{a+b})$


Bài viết đã được chỉnh sửa nội dung bởi PlanBbyFESN: 23-01-2016 - 16:48

:huh:


#54
Gachdptrai12

Gachdptrai12

    Thượng sĩ

  • Điều hành viên THCS
  • 280 Bài viết

cho a,b,c là các số thực dương chứng minh $(a+b+c)^{3}\geq \frac{27}{4}(a^{^{2}}b+b^{2}c+c^{2}a+abc)$


Bài viết đã được chỉnh sửa nội dung bởi Gachdptrai12: 22-01-2016 - 23:13


#55
Gachdptrai12

Gachdptrai12

    Thượng sĩ

  • Điều hành viên THCS
  • 280 Bài viết

bài 31 có thể dùng S.O.S và S.O.C bài này mình đăng kí nếu ko ra xóa bl của mình nhé tránh spam cho topic  :v :v :v :3 :3



#56
royal1534

royal1534

    Trung úy

  • Điều hành viên THCS
  • 773 Bài viết

 

 

Bài 32: $a,b,c>0$

CM: $2(\frac{a+b}{c}+\frac{b+c}{a}+\frac{c+a}{b})\geq \frac{a}{b+c}+\frac{b}{c+a}+\frac{c}{a+b}+\frac{9}{2}$

 

 

Eđit lại cái đề bài này cái.Dấu '=' không xảy ra tại $a=b=c$ @@



#57
I Love MC

I Love MC

    Đại úy

  • Thành viên nổi bật 2016
  • 1861 Bài viết

Old and new inequality - Titu Adresscu
File đã dịch .

File gửi kèm



#58
I Love MC

I Love MC

    Đại úy

  • Thành viên nổi bật 2016
  • 1861 Bài viết

Bài 33 :
Bài toán 1490 : Tạp chí Crux {Bất đẳng thức Garfunkel}
Tìm $k$ nhỏ nhất sao cho $\sum \frac{a}{\sqrt{b+c}} \le k.\sqrt{a+b+c}$ biết $a,b,c>0$



#59
quoccuonglqd

quoccuonglqd

    Thượng sĩ

  • Thành viên
  • 219 Bài viết

bài 31 có thể dùng S.O.S và S.O.C bài này mình đăng kí nếu ko ra xóa bl của mình nhé tránh spam cho topic  :v :v :v :3 :3

Xin lỗi,SOC là gì ạ



#60
royal1534

royal1534

    Trung úy

  • Điều hành viên THCS
  • 773 Bài viết

Xin lỗi,SOC là gì ạ

Đây






1 người đang xem chủ đề

0 thành viên, 1 khách, 0 thành viên ẩn danh